Đến nội dung

LangTu Mua Bui

LangTu Mua Bui

Đăng ký: 13-11-2015
Offline Đăng nhập: 04-01-2019 - 10:24
-----

#716595 CM: $\left \{ X\left ( \lambda _{1}...

Gửi bởi LangTu Mua Bui trong 15-10-2018 - 15:39

Đặt $ u_{1}=(1,2,1),u_{2}=(2,1,0)$

Xét pt  $ \left ( u_{1}+ \lambda _{1}u_{2} \right ) +a\left ( u_{1}+ \lambda _{2}u_{2} \right )=0 (1)$ nếu tồn tại a thì chúng phụ thuộc tt.

Mặt khác $u_{1},u_{2} $là độc lập tuyến tính trong $R^{3} $ nên $(a+1)u_{1}+(\lambda+a\lambda_{2})=0 $ khi và chỉ khi hay pt (1) =0 khi và chỉ khi $ a=-1,\lambda_{1}=lambda_{2} $ trái đk giả thiết nên pt k tồn tại hay 2 vector đó thể đltt.



#714733 Chứng minh: $det(J(Y,X))= (detA)^{n}.(detB)^{n}$

Gửi bởi LangTu Mua Bui trong 23-08-2018 - 23:25

Viết J(X,Y)=$\frac{\partial y_{i}}{\partial x_{i}}$ là không ổn vì dễ nhầm là phép cuộn .Để tránh nhầm lẫn nên viết lại thành $\frac{\partial y_{i}}{\partial x_{j}}$

Ở bài này nên biết đến tích tensor của 2 ma trận thì dễ hiểu hơn  $J=B^{T} \bigotimes  A$

Biến đổi từ đề bài ta được : $\Rightarrow  y_{ij}= \frac{\partial y_{ih}}{\partial x_{mn}}= \frac{\partial \left ( \sum_{j=1}^{n}\sum_{k=1}^{n}a_{ij}x_{jk}b_{kh} \right ) }{\partial x_{mn}}$

$y_{ij} $ Chỉ khác 0 khi mn trùng với jk .Suy ra ma trận Jacobi biến đổi là 1 ma trận $n^{2}\times n^{2}$

   

Đặt $c_{ij}=b_{ij} $ và  viết dạng ma trận khối 

 

                             $J=\begin{bmatrix} c_{11}A &c_{12}A&.. &c_{nn}A \\ c_{21}A & .. &...& .. \\ ... &...& ...\\ c_{n1}A & ..&.. & c_{nn}A \end{bmatrix}$

 

 

Do cách tính định thức là tổng các tính các hoán vị nên khi nhân các phần tử của $c_{ij}$ với A thì không làm thay đổi hoán vị
Gọi F là trường vector các ma trận vuông cấp n là cấp của mỗi khối , R là trường các ma trận vuông $n^{2} $ của ma trận J 
 

Điều này dẫn đến : $det J=det_{F}(det_{R} J) $

 

                                      $  =det_{F}\left (sum _{\pi \in S_{n}}(sgn \pi)c_{1\pi_{i_{1}}}c_{2\pi_{i_{2}}}...c_{n\pi_{i_{n}}}A^{n} \right )$

 

$det_{F}$ là tính định thức với dạng ma trận khối ,$det_{R}$ là tính định thức coi các ma trận thông thường bằng cách coi các  khối như phần tử vô hướng. 

 

 Do $A^{n} $là ma trận vuông cấp n                 $=det_{F}A^{n}(\left (sum _{\pi \in S_{n}}(sgn \pi)c_{1\pi_{i_{1}}}c_{2\pi_{i_{2}}}...c_{n\pi_{i_{n}}}\right )^{n}$

                                                                          $=detA^{n}detC^{n}$

                                                                          $=detA^{n}(detB^{T})^{n}$

                                                   

                                                                  $  \Rightarrow det J=(detA)^{n}(detB)^{n}$




#714632 $dimC(A) \ge n$

Gửi bởi LangTu Mua Bui trong 21-08-2018 - 13:35

C(A) là tập hợp các ma trận nên  mọi ma trận thuộc C(A) đều thuộc không gian chứa các ma trận vuông $ M_{n\times n }$ thuộc 
Chứng minh dimC(A)>n 
Với không gian các ma trận ta biết số chiều là $n^{2} $

Theo định lý số chiều :

Ta có $dim (Im F(X)) +dim(ker F(X)) =n^{2} $ Vì thế  yêu cầu bài toán tương đương với số chiều $ker F(X) >n $

 

Ta sẽ đi chứng minh dim$ Im(F(X)) <n^{2}-n $

 

Xét ánh xạ  F(X)=AX-XA với X và A là các ma trận vuông cấp n 

 

Ánh xạ F là 1 ánh xạ tuyến tính do nó thỏa mãn 2 tính chất 

 

 Thật vậy :$ F(M+N)=A(M+N)-(M+N)A=AM-MA+AN-NA=F(M)+F(N)$
            và $F(kX)=k(AX-XA)=kF(X) $

 

Bằng biển đổi thông thường  ta luôn có F(X) là 1 ma trận có các phần tử trên đường chéo chính đều bằng 0 với mọi ma trận X 
VÌ vậy số chiều của$ Im F(X) $luôn nhỏ hơn $n^{2}-n$
 Yêu cầu bái toán đã được chứng minh .




#714513 Chứng minh rằng các ma trận $M,N$ đồng dạng và các ma trận $P,...

Gửi bởi LangTu Mua Bui trong 18-08-2018 - 14:31

Hai ma trận đồng dạng với nhau cho chúng có cùng đa thức cực tiểu như vậy ta có thể chéo hóa ma trận đó về dạng jordan .Đây là cách tổng quát cho mọi ma trận cho dù ma trận đó k có cơ sở để chéo hóa được.

 

 Các ma trận đường chéo $ A=diag(a_{ii}) $ có đặc điểm thuận tiện khi lũy thừa vì thế một đa thức P(x) thì  P(A) là một ma trận đường chéo có dạng $diag(P(a_{ii})) $ nó đúng cho cả các ma trận khối .
.
Ta  có tính chất nếu đa thức P(A)=0 thì nó là chia hết cho $g_{A} $ đa thức cực tiểu của ma trận A  
Xét P(N) là đa thức tối thiểu thì $diag(P(A+B) ,P(A-B) )$ Vì thế P(N)=0 khi chỉ khi P(A+B)=0 ,P(A-B) =0 tức P(N) là bội chung của của 2 đa thức cực tiểu A+B và A-B. .
Do  $g_{N} $là đa thức cực tiểu vì vậy là đa thức có bậc bé nhất suy ra nó là  bội chung nhỏ nhất của 2 đa thức cực tiểu $g_{A+B} $và $g_{A-B} $ 

Bây h ta sẽ đi chứng minh ma trận M cũng có đa thức cực tiểu là bội chung nhỏ nhất của $g_{A+B}$ và $g_{A-B}$
Xét thấy 

$\begin{pmatrix} A & B\\ B&A \end{pmatrix}=\frac{1}{2}\begin{pmatrix} (A+B)+(A-B) & (A+B)-(A-B) \\ (A+B)-(A-B)& (A+B)+(A-B) \end{pmatrix}$

$\begin{pmatrix} A &B \\ B&A \end{pmatrix}^{2}=\frac{1}{2}\begin{pmatrix} (A+B)^{2}+(A-B)^{2} & (A+B)^{2}-(A-B)^{2} \\ (A+B)^{2}-(A-B)^{2}& (A+B)^{2}+(A-B)^{2} \end{pmatrix}$
.. ..
$P(M)=\frac{1}{2}\begin{pmatrix} P(A+B)+P(A-B) & P(A+B)-P(A-B)\\ P(A+B)-P(A-B)& P(A+B)+P(A-B) \end{pmatrix}$
 

Tương tự nếu $P(M)=0 $thì $P(A+B)+P(A-B)=0$ và $P(A+B)-P(A-B)=0$ điều đó có nghĩa là P(M)=0 khi và chỉ khi$ P(A+B) $và $P(A-B) $đều bằng 0.
Dẫn tới các đa thức thỏa mãn$ g(M)=0$ thì chúng là bội chung của các đa thức cực tiểu $(A+B) $và $(A-B) $

Nếu là đa thức cực tiểu số bậc nhỏ nhất thì nó là duy nhất và là bội chung nhỏ nhất của 2 đa thức cực tiểu của 2 ma trận $(A+B) $và $(A-B) .$

Vậy ta đã chứng minh 2 ma trận M và N cùng đa thức cực tiểu vì thế chúng đồng dạng với nhau.




#605591 câu 1 đề thi OLP giải tích 1997

Gửi bởi LangTu Mua Bui trong 27-12-2015 - 20:29

cho số thực A khác 0.xét hàm số $f(x)$ xác định;liên tục trên [0;$+\infty $] và thỏa mãn $\underset{x\rightarrow +\infty}{lim}f(x)=A$. tính

$\underset{n\rightarrow +\infty}{lim}\int_{0}^{1}f(nx)dx$ 

Đặt $nx=t\Rightarrow  \int_{0}^{1}f(nx)dx=\frac{\int_{0}^{n}f(n)dt}{n}$

$\Rightarrow \lim_{n\to\infty}\int_{0}^{1}f(nx)dx=\lim_{n\to\infty}\frac{\int_{0}^{n}f(n)dt}{n} (Lopitan)=\lim_{n\to\infty}f(n)=A$



#601708 Đề thi Olympic toán sinh viên ĐH BK HN 2013

Gửi bởi LangTu Mua Bui trong 05-12-2015 - 11:58

Câu 5 
$\left\{ \begin{array}{l} f'\left( x \right) > 0\\ f\left( {f'\left( x \right)} \right) = - f(x) \end{array} \right.$

Ta có $f(f'(x))=-f(x)$

Thay $ x=f'(x) \Rightarrow f(f'(f'(x))))=f(x)$ Do $f'(x)>0 \Rightarrow f'(f'(x))=x  (1)$

$ \Rightarrow  f(f'(x))=-f(x)$

Do hàm khả vi cấp 2 nên Ta  đạo hàm 2 vế theo x $ \Rightarrow f''(x)f'(f'(x))=-f'(x)(2)$
 
Từ $ (1)(2)f''(x).f'(f'(x))\Leftrightarrow f''(x)x+f'(x)=0$

$ \Rightarrow f(x)=a\ln{x}+b$ Thay ngược lại đề bài $\Rightarrow  a=b=1 \Rightarrow f(x)=\ln{x} $



#601683 Đề thi Olympic toán học sinh viên 2012 Đại Học BK Hà Nội

Gửi bởi LangTu Mua Bui trong 05-12-2015 - 01:21

Xét $g(x)=\int_{0}^{x}f(t)dt$ Do f(x) khả vi 2 lần nên g(x) khả vi 3 lần 

Ta có$ g'(x)=f(x) ..$
 
Khai triển maclaurank $\Rightarrow g(x)=g(0)+g(0)x+\frac{g'(0)x^{2}}{3}+\frac{g''(0+a(x-x_{0}))x^{3}}{6}$ (0<a<1)

Thay x=1 ta được $g(1)=\int_{0}^{1}=f(0)+\frac{f'(0)}{2}+\frac{f''(\theta (x))}{6}$ với $c=0<\theta (x)<1$ Ta đpcm 
 



#601682 Đề thi Olympic toán học sinh viên 2012 Đại Học BK Hà Nội

Gửi bởi LangTu Mua Bui trong 05-12-2015 - 01:10

Câu 1
$x_n=\underbrace{\sqrt[3]{6+\sqrt[3]{6+...+\sqrt[3]{6}}}} $

Ta có $2=\sqrt[3]{8}=\sqrt[3]{6+2}=\sqrt[3]{6+\sqrt[3]{8}}=\sqrt[3]{6+\sqrt[3]{6+\sqrt[3]{6+...}}}$

$\Rightarrow \lim_{n\to\infty}6^n(2-x^{n})=0$



#601388 Tuyển tập một số bài toán Olympic SV Giải tích

Gửi bởi LangTu Mua Bui trong 03-12-2015 - 14:52

 Câu :3

Đặt 
$a_{n}=\int_{0}^{1}f^{n}(x)dx $

$\Rightarrow \lim_{n \to \infty}\sqrt[n]{a_{n}}=\lim_{n \to \infty}e^{\frac{\ln{a_{n}}}{n}}=\lim_{n \to \infty}e^{\ln{\frac{a_{n+1}}{a_{n}}}}=\lim_{n \to \infty}\frac{a_{n+1}}{a_{n}} $

 

Như ta đã biết $ \int_{0}^{1}f^{n}(x)dx=\sum_{i=0}^{i=n}f^{n}(\frac{i}{n})$

 $\Rightarrow \frac{\int_{0}^{1}f^{n+1}(x)dx}{\int_{0}^{1}f^{n}(x)dx}=\lim_{n\to\infty}\frac{\sum_{i=0}^{i=n}f^{n+1}(\frac{i}{n})}{\sum_{i=0}^{i=n}f^{n}(\frac{i}{n})}$

Do hàm f(x) liên tục$[0;1] \Rightarrow $ tồn tại $x_{0} $ sao cho$ f(x_{0})=maxf(x) ;x_{0} \in [0;1]$

Theo quy tắc ngắt bỏ VCL$\Rightarrow \frac{\int_{0}^{1}f^{n+1}(x)dx}{\int_{0}^{1}f^{n}(x)dx}=\lim_{n\to\infty}\frac{\sum_{i=0}^{i=n}f^{n+1}(\frac{i}{n})}{\sum_{i=0}^{i=n}f^{n}(\frac{i}{n})}

=\lim_{n\to\infty}\dfrac{f^{n+1}(x_{0})}{f^{n}(x_{0})}=f(x_{0})=Maxf(x)$




#599331 $I=\lim_{n\rightarrow +\infty }\frac{...

Gửi bởi LangTu Mua Bui trong 20-11-2015 - 23:20

 
 We have $\lim_{n\to\infty}\sqrt[n]{a_{n}}=\lim_{n\to\infty}e^{\frac{\ln{a_{n}}}{n}} $

$=e^{\ln{a_{n+1}-\ln{a_{n}}}}(Stole)=e^{\ln{\frac{a_{n+1}}{a_{n}}}}=\lim_{n\to\infty}\frac{a_{n+1}}{a_{n}}$

$\Rightarrow \lim{n\to\infty}\frac{(n+1)^{n+1}.n!}{(n+1).n!.n^{n}}=\lim_{n\to\infty}(1+\frac{1}{n})^{n}=e$



#599240 Tính giá trị biểu thức $\arctan u+\arctan v+\arctan...

Gửi bởi LangTu Mua Bui trong 20-11-2015 - 16:15

Gọi $u,v,w$ là ba nghiệm của phương trình $x^3-10x+11=0$. Tính giá trị của biểu thức $$\arctan u+\arctan v+\arctan w$$.

Tạp chí KoMaL, Hungary.

Ta có đẳng thức quen thuộc $x=\tan{\arctan{x}}$

Biển đổi qua Ta có công thức dễ dàng  

$\Rightarrow \arctan{x}+\arctan{y}=\arctan{\tan{\left ( (\arctan{x})+(\arctan{y}) \right )}} =\arctan{\left ( \frac{\tan{(\arctan{x})}+\tan{(\arctan{y})}}{1+\tan{(\arctan{x})}\tan{(\arctan{y})}} \right )}$
 
Theo đl viet bậc cao 
$\Rightarrow \arctan{x}+\arctan{y}+\arctan{z}=\arctan{\frac{x+y+z+xyz}{1+xy+zx+yz}}$
$\left\{\begin{matrix} x+y+z=\frac{-b}{a}=0\\ xy+yz+zx=\frac{c}{a}=-10 \\zyz=\frac{d}{a}=11 \end{matrix}\right.$

$\arctan{x}+\arctan{y}+\arctan{z}=\arctan{\frac{x+y+z+xyz}{1+xy+zx+yz}}=\arctan{-\frac{11}{9}}$

 



#599230 $ \arcsin x +\arcsin x\sqrt{15}=\frac...

Gửi bởi LangTu Mua Bui trong 20-11-2015 - 15:04

Giải phương trình $$ \arcsin x +\arcsin (x\sqrt{15})=\frac{\pi}{2} $$

Tạp chí Komal, Hungary.

$\arcsin{x}+\arcsin{x\sqrt{15}}=\frac{\pi}{2}$
Đk :x>0 
Xét hàm $y=\arcsin{x}+\arccos{x} ;y'=0$  y là hàm hằng

$ \Rightarrow y=c $ thay giá trị bất kị vào để tìm c $;y(0)=c=\frac{\pi}{2}$


$\Rightarrow  \arcsin x +\arcsin (x\sqrt{15})=\frac{\pi}{2}$$ \Leftrightarrow \arcsin{x\sqrt{15}}=\arccos{x}$
 
 
Ta có$ t=\arcsin{sin{t}}=\arcsin{(\sqrt{1-\cos^{2}{t}})}$
 
$\arccos{x}$Ở đây Ta hiểu$ x=\cos{t} \Rightarrow \arccos{x}=t $

Ta có $\sin^{2}{t}+\cos^{2}{t}=1 \Rightarrow \sin{t}=\sqrt{1-\cos^{2}{t}}$ Ở đây là xét x>0  $\Rightarrow \arccos{x}=\arcsin{\sqrt{1-x^{2}}}$

$\Rightarrow 15x^{2}=1-x^{2} \Rightarrow x=\dfrac{1}{4}$



#598960 ĐỀ THI GIỮA KÌ MÔN GIẢI TÍCH 20151 (ĐHBKHN)

Gửi bởi LangTu Mua Bui trong 18-11-2015 - 16:59

Khai triển taylor tại $x=2015  \Rightarrow P(x)=P(2015)+\sum_{k=1}^{n}\frac{P^{k}(2015)(x-2015)^{k}}{k!}$

$P(x)=1+\sum_{k=1}^{n}(-1^{k})(x-2015)^{k}=1+\sum_{k=1}^{2015}(2015-x)^{k}$

$\Rightarrow P(2014)=1+2015=2016$




#598774 \int_{0}^{1}f(x).x^{n}=0$ với mọi...

Gửi bởi LangTu Mua Bui trong 17-11-2015 - 15:05

$ \Rightarrow \int_{0}^{1}C^{k}_{n}x^{n-k}m^{k}f(x)dx =0 $

$ \Rightarrow \left ( \sum_{k=0}^{n}C^{k}_{n}x^{n-k}m^{k}f(x) \right )=0 $

$\Leftrightarrow \int_{0}^{1}(m+x)^{n}f(x)dx \forall n\in N ;m\in R $

Đặt  $m+x=t \Rightarrow \int_{0}^{m+1}x^{n}f(x-m)dx=0 ;n=1 \Rightarrow \int_{0}^{m+1}f(x-m)dx=0 $;

Đặt $ g(m)=\int_{0}^{m+1}f(x-m)dx=0 \forall m \in R \Rightarrow g(m)=c \Rightarrow g'(x)=0 $ $ \Rightarrow f(1)=0 $




#598220 Chứng minh Xn= (1+1/2)(1+1/4)...(1/2^n) có giới hạn.

Gửi bởi LangTu Mua Bui trong 13-11-2015 - 23:07

$x_{n}=\prod_{k=1}^{n}(1+\frac{1}{2^{n}})=2.\left ( 1-\frac{1}{2} \right )\prod_{k=1}^{n}\left ( 1+\frac{1}{2^{k}} \right )=2(1-\frac{1}{2})(1+\frac{1}{2})(1+\frac{1}{4})..(1+\frac{1}{2^{n}})=2\left ( 1-\frac{1}{2^{2^{n}}} \right )\Rightarrow  $ Dãy số có giới hạn =2